2022 AMC 8 Problems/Problem 8

Revision as of 11:50, 28 January 2022 by Wamofan (talk | contribs) (Problem: added problem)

Problem

What is the value of \[\frac{1}{3}\cdot\frac{2}{4}\cdot\frac{3}{5}\cdots\frac{18}{20}\cdot\frac{19}{21}\cdot\frac{20}{22}?\]

$\textbf{(A)} ~\frac{1}{462}\qquad\textbf{(B)} ~\frac{1}{231}\qquad\textbf{(C)} ~\frac{1}{132}\qquad\textbf{(D)} ~\frac{2}{213}\qquad\textbf{(E)} ~\frac{1}{22}\qquad$

Solution

See Also

2022 AMC 8 (ProblemsAnswer KeyResources)
Preceded by
Problem 7
Followed by
Problem 9
1 2 3 4 5 6 7 8 9 10 11 12 13 14 15 16 17 18 19 20 21 22 23 24 25
All AJHSME/AMC 8 Problems and Solutions

The problems on this page are copyrighted by the Mathematical Association of America's American Mathematics Competitions. AMC logo.png